LSAT and Law School Admissions Forum

Get expert LSAT preparation and law school admissions advice from PowerScore Test Preparation.

User avatar
 Dave Killoran
PowerScore Staff
  • PowerScore Staff
  • Posts: 5852
  • Joined: Mar 25, 2011
|
#27289
Complete Question Explanation
(The complete setup for this game can be found here: lsat/viewtopic.php?t=1171)

The correct answer choice is (E)

The question stem creates the following conditional relationship:
  • L ..... :arrow: ..... P
This rule establishes that Z, which already has L, must also have P:
Oct 01_M12_game#2_L9_explanations_game#1_#8_diagram_1.png
As Z has power windows, Z cannot have a sunroof and answer choice (E) is correct.
You do not have the required permissions to view the files attached to this post.
 JordanW
  • Posts: 1
  • Joined: Jul 10, 2019
|
#66409
I understand why the answer choice is E, but I don't understand why A is wrong. T has power windows must be false, because if T has P than it must also have L according to the introduced rule of L --> P. If T has LP then T < Z rule is not satisfied.
 Malila Robinson
PowerScore Staff
  • PowerScore Staff
  • Posts: 296
  • Joined: Feb 01, 2018
|
#66434
Hi jwilia,
Careful, the rule says L --> P, but you have interpreted that to mean that L & P are always together. What the rule is saying is that if we know a car has L then it automatically gets P. But if we know a car has P we have no idea whether it will have L, that would be a Mistaken Reversal. So Answer A may be true, Car T might have P and no other options.
Hope that helps!
Malila

Get the most out of your LSAT Prep Plus subscription.

Analyze and track your performance with our Testing and Analytics Package.